Merge remote branch 'public/master'
[course.git] / latex / problems / Serway_and_Jewett_4 / problem19.59.tex
1 \begin{problem5}{19.59}
2 Two small spheres of mass $m$ are suspended from strings of length $l$
3 that are connected at a common point.  One sphere has charge $Q$, and
4 the other has charge $2Q$.  The strings make angles $\theta_1$ and
5 $\theta_2$ with the vertical.
6 \Part{a} How are $\theta_1$ and $\theta_2$ related?
7 \Part{b} Assume that $\theta_1$ and $\theta_2$ are small.
8  Show that the distance $r$ between the spheres is given by
9  \begin{equation}
10   r \approx \left( \frac{4 k_e Q^2 l}{mg} \right)^{1/3}
11  \end{equation}
12 \end{problem*} % problem 19.59
13
14 \empaddtoprelude{
15   pair A, B, C, D, mFe, mFg;
16   numeric L, theta, mFe, mFg;
17   L := 3cm; theta := 15;
18   mFe := 10pt; mFg := 20pt; % in the big picture (b)
19   A := L*dir(-90-theta);  B := (0cm,0cm);  C := L*dir(-90+theta); D := L*dir(-90);
20 }
21
22 \begin{solution}
23 \begin{center}
24 \begin{empfile}[5a]
25 \begin{emp}(0, 0)
26   pair Ae, At, pFe, pFg;
27   %Ap := A / cosd theta; % extend R as deep as the vertical
28   Ae := A + unitvector(A) * 4mFg / cosd theta; % extend R as deep as Fg
29   At := A + (unitvector(A) rotated -90) * 4mFe / cosd theta; % extend as far out as Fe
30   pFe := A-(4mFe,0);    % tip of electric force
31   pFg := A-(0,4mFg);    % top of gravitational force
32   dotlabel.bot(" ", B);
33   draw A -- B; % broken
34   draw B -- D dashed evenly;  % drop vertical from the pivot
35   label.bot(btex $\theta_1$ etex, draw_langle(A, B, D, .6L));
36   % draw forces
37   label.bot(btex $F_E$ etex, draw_force(C,A,4mFe));
38   label.bot(btex $F_g$ etex, draw_force(A+dir(90),A,4mFg));
39   % draw axes
40   drawarrow A -- Ae; % extend string radius
41   drawarrow A -- At; % draw tangential axis head
42   draw A -- (At rotatedabout(A, 180)); % and tangential axis tail
43   label.top(btex $x_t$ etex, At);
44   label.lft(btex $x_r$ etex, Ae);
45   % label force angles
46   label.lft(btex $\theta_1$ etex, draw_langle(At, A, pFe, 3mFe));  
47   label.bot(btex $\theta_1$ etex, draw_langle(Ae, A, pFg, .6L));  
48   draw_right_angle(At, A, Ae, .7mFe);
49   draw_right_angle(pFe, A, pFg, 1mFe);  
50   % draw charge
51   draw_pcharge(A, 4pt);
52 \end{emp}
53 \end{empfile}
54 \hspace{1cm}
55 \begin{empfile}[5b]
56 \begin{emp}(0, 0)
57   dotlabel.bot(" ", B);
58   draw A -- B -- C;
59   draw B -- D dashed evenly;
60   label.bot(btex $\theta_1$ etex, draw_langle(A, B, D, .6L));
61   label.bot(btex $\theta_2$ etex, draw_langle(D, B, C, .6L));
62   label.lft(btex $F_E$ etex, draw_force(C,A,mFe));
63   label.bot(btex $F_g$ etex, draw_force(A+dir(90),A,mFg));
64   draw_pcharge(A, 4pt);
65   draw_pcharge(C, 6pt);
66   labeloffset := 8pt;
67   label.rt(btex $m$ etex, A);
68   label.rt(btex $m$ etex, C);
69   label.ulft(btex $Q$ etex, A);
70   label.bot(btex $2Q$ etex, C);
71 \end{emp}
72 \end{empfile}
73 \end{center}
74 \Part{a}
75 Assuming that the charges are not rotating about each other, the
76 forces on each charge must cancel.  The forces on each sphere are
77 gravity $F_g = mg$, electrostatic $F_E = k_e 2Q^2/r^2$, and tension
78 $T$.  The tension will automatically handle canceling forces in the
79 radial direction, so we need only consider the tangential direction.
80 Let us assume that $F_E$ is purely in the horizontal direction
81 (see \Part{Note}).  Summing the tangential forces on the first sphere
82 \begin{align}
83  0 &= F_E \cos\theta_1 - F_g \sin\theta_1 \\
84  \tan\theta_1 &= \frac{F_E}{F_g}
85 \end{align}
86 And on the second sphere $\tan\theta_2 = \frac{F_E}{F_g}$
87  so $\theta_1 = \theta_2 = \theta$.
88
89 \Part{b}
90 \begin{align}
91  r &= 2 l \sin\theta
92     \approx 2 l \tan\theta
93     = 2 l \frac{F_E}{F_g}
94     = 2 l \frac{k_e 2 Q^2 / r^2}{mg} \\
95  r &\approx \left( \frac{4 l k_e Q^2}{mg} \right)^{1/3} \;,
96 \end{align}
97 where we used the small angle approximation
98 $\sin\theta\approx\tan\theta$ for small $\theta$.
99
100 \Part{Note} Why $\vect{F}_E$ is horizontal.
101
102 Let $q$ be the charge on the first mass and $Q$ be the charge on the
103 second.  The force of $1$ on $2$ is given by $F_{12} = k_e
104 qQ\rhat_{12}/r^2$.  This is identical to the force of $1$ on $2$ that
105 we would get if we had put $Q$ on $1$ and $q$ on $2$ (let us say ``the
106 electric force does not care about which mass has which charge'').
107 The only difference between the two masses is the charge, and the only
108 effect of that difference (the electrostatic force) does not care
109 about the difference, so the final situation must be symmetric
110 ($\theta_1 = \theta_2$ [no calculation required :p] and $\vect{r}$ is
111 horizontal).  Because $\vect{F_E} \propto \rhat_{12}$ it must also be
112 horizontal.
113 \end{solution}